Prove that $frac{k^7}{7}+frac{k^5}{5}+frac{2k^3}{3}-frac{k}{105}$ is an integer. [duplicate]












8












$begingroup$



This question already has an answer here:




  • Prove $frac{k^7}7+frac{k^5}5+frac{2k^3}3-frac k{105}$ is a integer by mathematical induction [duplicate]

    2 answers




Prove that $$frac{k^7}{7}+frac{k^5}{5}+frac{2k^3}{3}-frac{k}{105}$$ is an integer using mathematical induction.



I tried using mathematical induction but using binomial formula also it becomes little bit complicated.



Please show me your proof.



Sorry if this question was already asked. Actually i did not found it. In that case only sharing the link will be enough.










share|cite|improve this question











$endgroup$



marked as duplicate by Bill Dubuque algebra-precalculus
Users with the  algebra-precalculus badge can single-handedly close algebra-precalculus questions as duplicates and reopen them as needed.

StackExchange.ready(function() {
if (StackExchange.options.isMobile) return;

$('.dupe-hammer-message-hover:not(.hover-bound)').each(function() {
var $hover = $(this).addClass('hover-bound'),
$msg = $hover.siblings('.dupe-hammer-message');

$hover.hover(
function() {
$hover.showInfoMessage('', {
messageElement: $msg.clone().show(),
transient: false,
position: { my: 'bottom left', at: 'top center', offsetTop: -7 },
dismissable: false,
relativeToBody: true
});
},
function() {
StackExchange.helpers.removeMessages();
}
);
});
});
Jan 16 at 20:41


This question has been asked before and already has an answer. If those answers do not fully address your question, please ask a new question.














  • 2




    $begingroup$
    Put all terms on common denominator and prime factor the denominator and if possible numerator.
    $endgroup$
    – mathreadler
    Jan 16 at 17:00












  • $begingroup$
    It is $15k^7+21k^5+70k^3-k$
    $endgroup$
    – M Desmond
    Jan 16 at 17:01












  • $begingroup$
    You now need everything to be divisible by $105 = 3cdot 5cdot 7$
    $endgroup$
    – mathreadler
    Jan 16 at 17:05










  • $begingroup$
    Sorry ! I did not get what you want to point out. If you are saying about proving divisibility it is still a task : how to prove it by induction ? The p:k+1 will contain nearly 20 terms
    $endgroup$
    – M Desmond
    Jan 16 at 17:10








  • 1




    $begingroup$
    Read this marvelous answer on how to make an induction proof work
    $endgroup$
    – Ross Millikan
    Jan 16 at 17:11
















8












$begingroup$



This question already has an answer here:




  • Prove $frac{k^7}7+frac{k^5}5+frac{2k^3}3-frac k{105}$ is a integer by mathematical induction [duplicate]

    2 answers




Prove that $$frac{k^7}{7}+frac{k^5}{5}+frac{2k^3}{3}-frac{k}{105}$$ is an integer using mathematical induction.



I tried using mathematical induction but using binomial formula also it becomes little bit complicated.



Please show me your proof.



Sorry if this question was already asked. Actually i did not found it. In that case only sharing the link will be enough.










share|cite|improve this question











$endgroup$



marked as duplicate by Bill Dubuque algebra-precalculus
Users with the  algebra-precalculus badge can single-handedly close algebra-precalculus questions as duplicates and reopen them as needed.

StackExchange.ready(function() {
if (StackExchange.options.isMobile) return;

$('.dupe-hammer-message-hover:not(.hover-bound)').each(function() {
var $hover = $(this).addClass('hover-bound'),
$msg = $hover.siblings('.dupe-hammer-message');

$hover.hover(
function() {
$hover.showInfoMessage('', {
messageElement: $msg.clone().show(),
transient: false,
position: { my: 'bottom left', at: 'top center', offsetTop: -7 },
dismissable: false,
relativeToBody: true
});
},
function() {
StackExchange.helpers.removeMessages();
}
);
});
});
Jan 16 at 20:41


This question has been asked before and already has an answer. If those answers do not fully address your question, please ask a new question.














  • 2




    $begingroup$
    Put all terms on common denominator and prime factor the denominator and if possible numerator.
    $endgroup$
    – mathreadler
    Jan 16 at 17:00












  • $begingroup$
    It is $15k^7+21k^5+70k^3-k$
    $endgroup$
    – M Desmond
    Jan 16 at 17:01












  • $begingroup$
    You now need everything to be divisible by $105 = 3cdot 5cdot 7$
    $endgroup$
    – mathreadler
    Jan 16 at 17:05










  • $begingroup$
    Sorry ! I did not get what you want to point out. If you are saying about proving divisibility it is still a task : how to prove it by induction ? The p:k+1 will contain nearly 20 terms
    $endgroup$
    – M Desmond
    Jan 16 at 17:10








  • 1




    $begingroup$
    Read this marvelous answer on how to make an induction proof work
    $endgroup$
    – Ross Millikan
    Jan 16 at 17:11














8












8








8


1



$begingroup$



This question already has an answer here:




  • Prove $frac{k^7}7+frac{k^5}5+frac{2k^3}3-frac k{105}$ is a integer by mathematical induction [duplicate]

    2 answers




Prove that $$frac{k^7}{7}+frac{k^5}{5}+frac{2k^3}{3}-frac{k}{105}$$ is an integer using mathematical induction.



I tried using mathematical induction but using binomial formula also it becomes little bit complicated.



Please show me your proof.



Sorry if this question was already asked. Actually i did not found it. In that case only sharing the link will be enough.










share|cite|improve this question











$endgroup$





This question already has an answer here:




  • Prove $frac{k^7}7+frac{k^5}5+frac{2k^3}3-frac k{105}$ is a integer by mathematical induction [duplicate]

    2 answers




Prove that $$frac{k^7}{7}+frac{k^5}{5}+frac{2k^3}{3}-frac{k}{105}$$ is an integer using mathematical induction.



I tried using mathematical induction but using binomial formula also it becomes little bit complicated.



Please show me your proof.



Sorry if this question was already asked. Actually i did not found it. In that case only sharing the link will be enough.





This question already has an answer here:




  • Prove $frac{k^7}7+frac{k^5}5+frac{2k^3}3-frac k{105}$ is a integer by mathematical induction [duplicate]

    2 answers








algebra-precalculus proof-writing induction






share|cite|improve this question















share|cite|improve this question













share|cite|improve this question




share|cite|improve this question








edited Jan 16 at 17:00







M Desmond

















asked Jan 16 at 16:59









M DesmondM Desmond

1877




1877




marked as duplicate by Bill Dubuque algebra-precalculus
Users with the  algebra-precalculus badge can single-handedly close algebra-precalculus questions as duplicates and reopen them as needed.

StackExchange.ready(function() {
if (StackExchange.options.isMobile) return;

$('.dupe-hammer-message-hover:not(.hover-bound)').each(function() {
var $hover = $(this).addClass('hover-bound'),
$msg = $hover.siblings('.dupe-hammer-message');

$hover.hover(
function() {
$hover.showInfoMessage('', {
messageElement: $msg.clone().show(),
transient: false,
position: { my: 'bottom left', at: 'top center', offsetTop: -7 },
dismissable: false,
relativeToBody: true
});
},
function() {
StackExchange.helpers.removeMessages();
}
);
});
});
Jan 16 at 20:41


This question has been asked before and already has an answer. If those answers do not fully address your question, please ask a new question.









marked as duplicate by Bill Dubuque algebra-precalculus
Users with the  algebra-precalculus badge can single-handedly close algebra-precalculus questions as duplicates and reopen them as needed.

StackExchange.ready(function() {
if (StackExchange.options.isMobile) return;

$('.dupe-hammer-message-hover:not(.hover-bound)').each(function() {
var $hover = $(this).addClass('hover-bound'),
$msg = $hover.siblings('.dupe-hammer-message');

$hover.hover(
function() {
$hover.showInfoMessage('', {
messageElement: $msg.clone().show(),
transient: false,
position: { my: 'bottom left', at: 'top center', offsetTop: -7 },
dismissable: false,
relativeToBody: true
});
},
function() {
StackExchange.helpers.removeMessages();
}
);
});
});
Jan 16 at 20:41


This question has been asked before and already has an answer. If those answers do not fully address your question, please ask a new question.










  • 2




    $begingroup$
    Put all terms on common denominator and prime factor the denominator and if possible numerator.
    $endgroup$
    – mathreadler
    Jan 16 at 17:00












  • $begingroup$
    It is $15k^7+21k^5+70k^3-k$
    $endgroup$
    – M Desmond
    Jan 16 at 17:01












  • $begingroup$
    You now need everything to be divisible by $105 = 3cdot 5cdot 7$
    $endgroup$
    – mathreadler
    Jan 16 at 17:05










  • $begingroup$
    Sorry ! I did not get what you want to point out. If you are saying about proving divisibility it is still a task : how to prove it by induction ? The p:k+1 will contain nearly 20 terms
    $endgroup$
    – M Desmond
    Jan 16 at 17:10








  • 1




    $begingroup$
    Read this marvelous answer on how to make an induction proof work
    $endgroup$
    – Ross Millikan
    Jan 16 at 17:11














  • 2




    $begingroup$
    Put all terms on common denominator and prime factor the denominator and if possible numerator.
    $endgroup$
    – mathreadler
    Jan 16 at 17:00












  • $begingroup$
    It is $15k^7+21k^5+70k^3-k$
    $endgroup$
    – M Desmond
    Jan 16 at 17:01












  • $begingroup$
    You now need everything to be divisible by $105 = 3cdot 5cdot 7$
    $endgroup$
    – mathreadler
    Jan 16 at 17:05










  • $begingroup$
    Sorry ! I did not get what you want to point out. If you are saying about proving divisibility it is still a task : how to prove it by induction ? The p:k+1 will contain nearly 20 terms
    $endgroup$
    – M Desmond
    Jan 16 at 17:10








  • 1




    $begingroup$
    Read this marvelous answer on how to make an induction proof work
    $endgroup$
    – Ross Millikan
    Jan 16 at 17:11








2




2




$begingroup$
Put all terms on common denominator and prime factor the denominator and if possible numerator.
$endgroup$
– mathreadler
Jan 16 at 17:00






$begingroup$
Put all terms on common denominator and prime factor the denominator and if possible numerator.
$endgroup$
– mathreadler
Jan 16 at 17:00














$begingroup$
It is $15k^7+21k^5+70k^3-k$
$endgroup$
– M Desmond
Jan 16 at 17:01






$begingroup$
It is $15k^7+21k^5+70k^3-k$
$endgroup$
– M Desmond
Jan 16 at 17:01














$begingroup$
You now need everything to be divisible by $105 = 3cdot 5cdot 7$
$endgroup$
– mathreadler
Jan 16 at 17:05




$begingroup$
You now need everything to be divisible by $105 = 3cdot 5cdot 7$
$endgroup$
– mathreadler
Jan 16 at 17:05












$begingroup$
Sorry ! I did not get what you want to point out. If you are saying about proving divisibility it is still a task : how to prove it by induction ? The p:k+1 will contain nearly 20 terms
$endgroup$
– M Desmond
Jan 16 at 17:10






$begingroup$
Sorry ! I did not get what you want to point out. If you are saying about proving divisibility it is still a task : how to prove it by induction ? The p:k+1 will contain nearly 20 terms
$endgroup$
– M Desmond
Jan 16 at 17:10






1




1




$begingroup$
Read this marvelous answer on how to make an induction proof work
$endgroup$
– Ross Millikan
Jan 16 at 17:11




$begingroup$
Read this marvelous answer on how to make an induction proof work
$endgroup$
– Ross Millikan
Jan 16 at 17:11










8 Answers
8






active

oldest

votes


















5












$begingroup$

@I like Serena has a great answer but since the OP asked for a proof by induction, I'll show what that would look like. Define
$$f(k)=frac{k^7}{7}+frac{k^5}{5}+frac{2k^3}{3}-frac{k}{105}=frac{15k^7 + 21k^5+70k^3-k}{105}$$



For our base case, let $k=1$. Then we have
$$f(1)=frac{15+21+70-1}{105}=1$$
which is an integer. Now suppose $f(k)$ is an integer for some $kgeq 1$. We want to prove that $f(k+1)$ is also an integer. To that end, observe that
begin{align}
f(k+1)&=frac{15(k+1)^7 + 21(k+1)^5+70(k+1)^3-(k+1)}{105}\
&=frac{15k^7 + 105k^6+336k^5+630k^4 + 805k^3+735k^2+419k+105}{105}
end{align}

Therefore
begin{align}
f(k+1)-f(k)&=frac{105k^6+315k^5+630k^4+735k^3+735k^2+420k+105}{105}\
&=frac{105(k^6+3k^5+6k^4+7k^3+7k^2+4k+1)}{105}\
&= k^6+3k^5+6k^4+7k^3+7k^2+4k+1
end{align}

Which is an integer, say $N$. Rearranging this gives $f(k+1)=f(k)+N$ and since $f(k)$ is assumed to be an integer from the induction hypothesis, $f(k+1)$ is the sum of two integers, hence an integer.






share|cite|improve this answer









$endgroup$













  • $begingroup$
    Thanks 4 your effort
    $endgroup$
    – M Desmond
    Jan 16 at 17:41



















10












$begingroup$

We have:
$$frac{k^7}{7}+frac{k^5}{5}+frac{2k^3}{3}-frac{k}{105}
=frac{15k^7+21k^5+70k^3-k}{3cdot 5cdot 7}
$$

To prove this is an integer we need that:
$$15k^7+21k^5+70k^3-kequiv 0 pmod{3cdot 5cdot 7}$$
According to the Chinese Remainder Theorem, this is the case iff
$$begin{cases}15k^7+21k^5+70k^3-kequiv 0 pmod{3} \
15k^7+21k^5+70k^3-kequiv 0 pmod{5}\
15k^7+21k^5+70k^3-kequiv 0 pmod{7}end{cases} iff
begin{cases}k^3-kequiv 0 pmod{3} \
k^5-kequiv 0 pmod{5}\
k^7-kequiv 0 pmod{7}end{cases}$$

Fermat's Little Theorem says that $k^pequiv k pmod{p}$ for any prime $p$ and integer $k$.



Therefore the original expression is an integer.






share|cite|improve this answer











$endgroup$









  • 1




    $begingroup$
    Nice. You are a fast writer!
    $endgroup$
    – mathreadler
    Jan 16 at 17:12






  • 1




    $begingroup$
    Excellent answer!
    $endgroup$
    – manooooh
    Jan 16 at 17:15










  • $begingroup$
    Fermat's Little Theorem says that if $p$ is prime and $k$ is an integer, then we have $k^p=k$ (mod $p$). How does it follow from $k^p=k$ (mod $p$) that $k$ is an integer?
    $endgroup$
    – EuxhenH
    Jan 16 at 17:19










  • $begingroup$
    It doesn't @EuxhenH. I just stated the result first and the conditions afterwards. It's exactly how you say it is.
    $endgroup$
    – Klaas van Aarsen
    Jan 16 at 17:22










  • $begingroup$
    @EuxhenH an integer raised to an integer is always an integer because integers are closed under multiplication and addition.
    $endgroup$
    – mathreadler
    Jan 16 at 17:22





















2












$begingroup$

Call the expression $f(k)$. As it's a degree $7$ polynomial, it obeys the recurrence
$$sum_{j=0}^8(-1)^jbinom8jf(k-j)=0.$$
Thus
$$f(k)=8f(k-1)-28f(k-2)+56f(k-3)-70f(k-4)+56f(k-5)-28f(k-6)+8f(k-7)-f(k-8)$$
so that if $f$ takes eight consecutive integer values, by induction, all subsequent
values are integers too.






share|cite|improve this answer









$endgroup$













  • $begingroup$
    Very interesting. What causes the recurrence equation?
    $endgroup$
    – mathreadler
    Jan 16 at 17:20










  • $begingroup$
    @mathreadler iterated differences.
    $endgroup$
    – Lord Shark the Unknown
    Jan 16 at 17:21










  • $begingroup$
    Does it work only with 7 degree ? Please give a reference where i can find this relations !
    $endgroup$
    – M Desmond
    Jan 16 at 17:31








  • 1




    $begingroup$
    @MDesmond $Delta f(k) := f(k+1)-f(k), $ reduces the degree of the polynomial $,f(k),$ since leading terms cancel. So if $f$ has degree $n$ then $,Delta^{n+1} f(k) = 0,$ yields a recurrence for $f(k).,$ Above is the binomial expansion using $,Delta = S-1,$ where $,S,f(k) = f(k+1),$ is the shift operator. See any textbook that treats recurrences or finite differences.
    $endgroup$
    – Bill Dubuque
    Jan 16 at 20:01



















2












$begingroup$

hint...if you only want to use induction, let $$f(k)=15k^7+21k^5+70k^3-k$$
and consider $$f(k+1)-f(k)=$$



For the induction step you have to show this is divisible by $105$



So, for example, $$(k+1)^7-k^7=7N+1$$ where $N$ is an integer, etc...



Can you finish?






share|cite|improve this answer









$endgroup$





















    2












    $begingroup$

    You can use the binomial transform to prove that



    $$frac{k^7}{7}+frac{k^5}{5}+frac{2k^3}{3}-frac{k}{105}
    \={kchoose1}+28{kchoose2}+292{kchoose3}+1248{kchoose4}+2424{kchoose5}+2160{kchoose6}+720{kchoose7}$$






    share|cite|improve this answer









    $endgroup$





















      0












      $begingroup$

      Base case for $k=1$: $$frac{1^7}{7}+frac{1^5}{5}+frac{2*1^3}{3}-frac{1}{105}=1$$



      Now, assume for some k that $frac{k^7}{7}+frac{k^5}{5}+frac{2k^3}{3}-frac{k}{105}$ is indeed in integer.



      Then $$frac{(k+1)^7}{7}+frac{(k+1)^5}{5}+frac{2(k+1)^3}{3}-frac{k+1}{105}=\frac{sum_{i=0}^7binom{7}{i}k^i}{7}+frac{sum_{i=0}^5binom{5}{i}k^i}{5}+2frac{sum_{i=0}^3binom{3}{i}k^i}{3}-frac{k+1}{105}=$$



      Extracting the highest indexed term from each sum (and the $-frac{k}{105}$ at the end): $$frac{k^7}{7}+frac{k^5}{5}+frac{2k^3}{3}-frac{k}{105}+frac{sum_{i=0}^6binom{7}{i}k^i}{7}+frac{sum_{i=0}^4binom{5}{i}k^i}{5}+2frac{sum_{i=0}^2binom{3}{i}k^i}{3}-frac{1}{105}$$



      By the induction hypothesis, the sum of the first four terms is an integer so, if we can show the rest of the above sum is an integer, we will be done. Use the fact that, for any prime, $p$, $p|binom{p}{k}$ where $1leq kleq p-1$. This is because $$binom{p}{k}=frac{p(p-1)...(p-k+1)}{k(k-1)...1}$$



      $p$ divides the numerator but not the denominator (as $1leq kleq p-1$) so $p|binom{p}{k}$



      So each term in the remaining sum with index $i$ $geq1$ and $leq p-1$ ($p$ being the respective prime in each sum) is divisible by the corresponding $p$ in the denominator and produces an integer. The only non-integer terms left will be the ones at $i=0$, i.e. $$frac{binom{7}{0}k^0}{7}+frac{binom{5}{0}k^0}{5}+frac{2binom{3}{0}k^0}{3}-frac{1}{105}=frac{1}{7}+frac{1}{5}+frac{2}{3}-frac{1}{105}=1$$



      So $frac{(k+1)^7}{7}+frac{(k+1)^5}{5}+frac{2(k+1)^3}{3}-frac{k+1}{105}$ is a sum of integers making it an integer.






      share|cite|improve this answer









      $endgroup$





















        0












        $begingroup$

        Because
        $$frac{k^7}{7}+frac{k^5}{5}+frac{2k^3}{3}-frac{k}{105}=frac{k^7-k}{7}+frac{k^5-k}{5}+frac{2(k^3-k)}{3}+k.$$






        share|cite|improve this answer









        $endgroup$













        • $begingroup$
          Same as my answer after dividing by $,3cdot 5cdot 7 $
          $endgroup$
          – Bill Dubuque
          Jan 16 at 19:18












        • $begingroup$
          They are similar but my equality we can see immediately. See please better that I wrote.
          $endgroup$
          – Michael Rozenberg
          Jan 16 at 19:20










        • $begingroup$
          In my experience most students don't "immediately" see such fraction expansions.. Rather they first put it over a common denominator, as I did. From that it is easy to read off the fraction expansion (but there is no need to rewrite the key (Fernat) divisibilities in fraction form).
          $endgroup$
          – Bill Dubuque
          Jan 16 at 19:28












        • $begingroup$
          I think you see that $-frac{1}{105}=-frac{1}{7}-frac{1}{5}-frac{2}{3}+1$. If so, we are done! I think, it's much more better than your writing.
          $endgroup$
          – Michael Rozenberg
          Jan 16 at 19:31












        • $begingroup$
          How do you propose that one "sees" things like that in general?
          $endgroup$
          – Bill Dubuque
          Jan 16 at 19:32



















        0












        $begingroup$

        Hint $ $ Note that $ 3!cdot!5!cdot!7mid overbrace{3!cdot! 5, (color{#c00}{k^7!-!k})+ 3!cdot! 7, (color{#c00}{k^5!-!k})- 5!cdot! 7 (color{#c00}{k^3!-!k})+ 3!cdot! 5cdot! 7, k^3}^{Large{rm sum = this/(3cdot 5cdot 7)}}, $ by $,rmoverbrace{little color{#c00}{Fermat}}^{Large p mid color{#c00}{k^p-k}}$



        Remark $ $ More generally this shows that if $,p,q,r,$ are primes and $,a,b,c,k,$ are integers



        $$quad pqr,mid, aqr,(k^p!-!k)+bpr,(k^q!-!k)+cpq,(k^r!-!k)$$






        share|cite|improve this answer











        $endgroup$













        • $begingroup$
          Note that we don't actually have to recognize the above form to prove it is $equiv 0pmod{p},$ for $,p = 3,5,7,,$ since simply computing it $!bmod p,$ using $,k^pequiv k,$ easily proves it is $equiv 0, $ But I showed the form of the expression to reveal how it was constructed.
          $endgroup$
          – Bill Dubuque
          Jan 16 at 19:35


















        8 Answers
        8






        active

        oldest

        votes








        8 Answers
        8






        active

        oldest

        votes









        active

        oldest

        votes






        active

        oldest

        votes









        5












        $begingroup$

        @I like Serena has a great answer but since the OP asked for a proof by induction, I'll show what that would look like. Define
        $$f(k)=frac{k^7}{7}+frac{k^5}{5}+frac{2k^3}{3}-frac{k}{105}=frac{15k^7 + 21k^5+70k^3-k}{105}$$



        For our base case, let $k=1$. Then we have
        $$f(1)=frac{15+21+70-1}{105}=1$$
        which is an integer. Now suppose $f(k)$ is an integer for some $kgeq 1$. We want to prove that $f(k+1)$ is also an integer. To that end, observe that
        begin{align}
        f(k+1)&=frac{15(k+1)^7 + 21(k+1)^5+70(k+1)^3-(k+1)}{105}\
        &=frac{15k^7 + 105k^6+336k^5+630k^4 + 805k^3+735k^2+419k+105}{105}
        end{align}

        Therefore
        begin{align}
        f(k+1)-f(k)&=frac{105k^6+315k^5+630k^4+735k^3+735k^2+420k+105}{105}\
        &=frac{105(k^6+3k^5+6k^4+7k^3+7k^2+4k+1)}{105}\
        &= k^6+3k^5+6k^4+7k^3+7k^2+4k+1
        end{align}

        Which is an integer, say $N$. Rearranging this gives $f(k+1)=f(k)+N$ and since $f(k)$ is assumed to be an integer from the induction hypothesis, $f(k+1)$ is the sum of two integers, hence an integer.






        share|cite|improve this answer









        $endgroup$













        • $begingroup$
          Thanks 4 your effort
          $endgroup$
          – M Desmond
          Jan 16 at 17:41
















        5












        $begingroup$

        @I like Serena has a great answer but since the OP asked for a proof by induction, I'll show what that would look like. Define
        $$f(k)=frac{k^7}{7}+frac{k^5}{5}+frac{2k^3}{3}-frac{k}{105}=frac{15k^7 + 21k^5+70k^3-k}{105}$$



        For our base case, let $k=1$. Then we have
        $$f(1)=frac{15+21+70-1}{105}=1$$
        which is an integer. Now suppose $f(k)$ is an integer for some $kgeq 1$. We want to prove that $f(k+1)$ is also an integer. To that end, observe that
        begin{align}
        f(k+1)&=frac{15(k+1)^7 + 21(k+1)^5+70(k+1)^3-(k+1)}{105}\
        &=frac{15k^7 + 105k^6+336k^5+630k^4 + 805k^3+735k^2+419k+105}{105}
        end{align}

        Therefore
        begin{align}
        f(k+1)-f(k)&=frac{105k^6+315k^5+630k^4+735k^3+735k^2+420k+105}{105}\
        &=frac{105(k^6+3k^5+6k^4+7k^3+7k^2+4k+1)}{105}\
        &= k^6+3k^5+6k^4+7k^3+7k^2+4k+1
        end{align}

        Which is an integer, say $N$. Rearranging this gives $f(k+1)=f(k)+N$ and since $f(k)$ is assumed to be an integer from the induction hypothesis, $f(k+1)$ is the sum of two integers, hence an integer.






        share|cite|improve this answer









        $endgroup$













        • $begingroup$
          Thanks 4 your effort
          $endgroup$
          – M Desmond
          Jan 16 at 17:41














        5












        5








        5





        $begingroup$

        @I like Serena has a great answer but since the OP asked for a proof by induction, I'll show what that would look like. Define
        $$f(k)=frac{k^7}{7}+frac{k^5}{5}+frac{2k^3}{3}-frac{k}{105}=frac{15k^7 + 21k^5+70k^3-k}{105}$$



        For our base case, let $k=1$. Then we have
        $$f(1)=frac{15+21+70-1}{105}=1$$
        which is an integer. Now suppose $f(k)$ is an integer for some $kgeq 1$. We want to prove that $f(k+1)$ is also an integer. To that end, observe that
        begin{align}
        f(k+1)&=frac{15(k+1)^7 + 21(k+1)^5+70(k+1)^3-(k+1)}{105}\
        &=frac{15k^7 + 105k^6+336k^5+630k^4 + 805k^3+735k^2+419k+105}{105}
        end{align}

        Therefore
        begin{align}
        f(k+1)-f(k)&=frac{105k^6+315k^5+630k^4+735k^3+735k^2+420k+105}{105}\
        &=frac{105(k^6+3k^5+6k^4+7k^3+7k^2+4k+1)}{105}\
        &= k^6+3k^5+6k^4+7k^3+7k^2+4k+1
        end{align}

        Which is an integer, say $N$. Rearranging this gives $f(k+1)=f(k)+N$ and since $f(k)$ is assumed to be an integer from the induction hypothesis, $f(k+1)$ is the sum of two integers, hence an integer.






        share|cite|improve this answer









        $endgroup$



        @I like Serena has a great answer but since the OP asked for a proof by induction, I'll show what that would look like. Define
        $$f(k)=frac{k^7}{7}+frac{k^5}{5}+frac{2k^3}{3}-frac{k}{105}=frac{15k^7 + 21k^5+70k^3-k}{105}$$



        For our base case, let $k=1$. Then we have
        $$f(1)=frac{15+21+70-1}{105}=1$$
        which is an integer. Now suppose $f(k)$ is an integer for some $kgeq 1$. We want to prove that $f(k+1)$ is also an integer. To that end, observe that
        begin{align}
        f(k+1)&=frac{15(k+1)^7 + 21(k+1)^5+70(k+1)^3-(k+1)}{105}\
        &=frac{15k^7 + 105k^6+336k^5+630k^4 + 805k^3+735k^2+419k+105}{105}
        end{align}

        Therefore
        begin{align}
        f(k+1)-f(k)&=frac{105k^6+315k^5+630k^4+735k^3+735k^2+420k+105}{105}\
        &=frac{105(k^6+3k^5+6k^4+7k^3+7k^2+4k+1)}{105}\
        &= k^6+3k^5+6k^4+7k^3+7k^2+4k+1
        end{align}

        Which is an integer, say $N$. Rearranging this gives $f(k+1)=f(k)+N$ and since $f(k)$ is assumed to be an integer from the induction hypothesis, $f(k+1)$ is the sum of two integers, hence an integer.







        share|cite|improve this answer












        share|cite|improve this answer



        share|cite|improve this answer










        answered Jan 16 at 17:36









        pwerthpwerth

        3,310417




        3,310417












        • $begingroup$
          Thanks 4 your effort
          $endgroup$
          – M Desmond
          Jan 16 at 17:41


















        • $begingroup$
          Thanks 4 your effort
          $endgroup$
          – M Desmond
          Jan 16 at 17:41
















        $begingroup$
        Thanks 4 your effort
        $endgroup$
        – M Desmond
        Jan 16 at 17:41




        $begingroup$
        Thanks 4 your effort
        $endgroup$
        – M Desmond
        Jan 16 at 17:41











        10












        $begingroup$

        We have:
        $$frac{k^7}{7}+frac{k^5}{5}+frac{2k^3}{3}-frac{k}{105}
        =frac{15k^7+21k^5+70k^3-k}{3cdot 5cdot 7}
        $$

        To prove this is an integer we need that:
        $$15k^7+21k^5+70k^3-kequiv 0 pmod{3cdot 5cdot 7}$$
        According to the Chinese Remainder Theorem, this is the case iff
        $$begin{cases}15k^7+21k^5+70k^3-kequiv 0 pmod{3} \
        15k^7+21k^5+70k^3-kequiv 0 pmod{5}\
        15k^7+21k^5+70k^3-kequiv 0 pmod{7}end{cases} iff
        begin{cases}k^3-kequiv 0 pmod{3} \
        k^5-kequiv 0 pmod{5}\
        k^7-kequiv 0 pmod{7}end{cases}$$

        Fermat's Little Theorem says that $k^pequiv k pmod{p}$ for any prime $p$ and integer $k$.



        Therefore the original expression is an integer.






        share|cite|improve this answer











        $endgroup$









        • 1




          $begingroup$
          Nice. You are a fast writer!
          $endgroup$
          – mathreadler
          Jan 16 at 17:12






        • 1




          $begingroup$
          Excellent answer!
          $endgroup$
          – manooooh
          Jan 16 at 17:15










        • $begingroup$
          Fermat's Little Theorem says that if $p$ is prime and $k$ is an integer, then we have $k^p=k$ (mod $p$). How does it follow from $k^p=k$ (mod $p$) that $k$ is an integer?
          $endgroup$
          – EuxhenH
          Jan 16 at 17:19










        • $begingroup$
          It doesn't @EuxhenH. I just stated the result first and the conditions afterwards. It's exactly how you say it is.
          $endgroup$
          – Klaas van Aarsen
          Jan 16 at 17:22










        • $begingroup$
          @EuxhenH an integer raised to an integer is always an integer because integers are closed under multiplication and addition.
          $endgroup$
          – mathreadler
          Jan 16 at 17:22


















        10












        $begingroup$

        We have:
        $$frac{k^7}{7}+frac{k^5}{5}+frac{2k^3}{3}-frac{k}{105}
        =frac{15k^7+21k^5+70k^3-k}{3cdot 5cdot 7}
        $$

        To prove this is an integer we need that:
        $$15k^7+21k^5+70k^3-kequiv 0 pmod{3cdot 5cdot 7}$$
        According to the Chinese Remainder Theorem, this is the case iff
        $$begin{cases}15k^7+21k^5+70k^3-kequiv 0 pmod{3} \
        15k^7+21k^5+70k^3-kequiv 0 pmod{5}\
        15k^7+21k^5+70k^3-kequiv 0 pmod{7}end{cases} iff
        begin{cases}k^3-kequiv 0 pmod{3} \
        k^5-kequiv 0 pmod{5}\
        k^7-kequiv 0 pmod{7}end{cases}$$

        Fermat's Little Theorem says that $k^pequiv k pmod{p}$ for any prime $p$ and integer $k$.



        Therefore the original expression is an integer.






        share|cite|improve this answer











        $endgroup$









        • 1




          $begingroup$
          Nice. You are a fast writer!
          $endgroup$
          – mathreadler
          Jan 16 at 17:12






        • 1




          $begingroup$
          Excellent answer!
          $endgroup$
          – manooooh
          Jan 16 at 17:15










        • $begingroup$
          Fermat's Little Theorem says that if $p$ is prime and $k$ is an integer, then we have $k^p=k$ (mod $p$). How does it follow from $k^p=k$ (mod $p$) that $k$ is an integer?
          $endgroup$
          – EuxhenH
          Jan 16 at 17:19










        • $begingroup$
          It doesn't @EuxhenH. I just stated the result first and the conditions afterwards. It's exactly how you say it is.
          $endgroup$
          – Klaas van Aarsen
          Jan 16 at 17:22










        • $begingroup$
          @EuxhenH an integer raised to an integer is always an integer because integers are closed under multiplication and addition.
          $endgroup$
          – mathreadler
          Jan 16 at 17:22
















        10












        10








        10





        $begingroup$

        We have:
        $$frac{k^7}{7}+frac{k^5}{5}+frac{2k^3}{3}-frac{k}{105}
        =frac{15k^7+21k^5+70k^3-k}{3cdot 5cdot 7}
        $$

        To prove this is an integer we need that:
        $$15k^7+21k^5+70k^3-kequiv 0 pmod{3cdot 5cdot 7}$$
        According to the Chinese Remainder Theorem, this is the case iff
        $$begin{cases}15k^7+21k^5+70k^3-kequiv 0 pmod{3} \
        15k^7+21k^5+70k^3-kequiv 0 pmod{5}\
        15k^7+21k^5+70k^3-kequiv 0 pmod{7}end{cases} iff
        begin{cases}k^3-kequiv 0 pmod{3} \
        k^5-kequiv 0 pmod{5}\
        k^7-kequiv 0 pmod{7}end{cases}$$

        Fermat's Little Theorem says that $k^pequiv k pmod{p}$ for any prime $p$ and integer $k$.



        Therefore the original expression is an integer.






        share|cite|improve this answer











        $endgroup$



        We have:
        $$frac{k^7}{7}+frac{k^5}{5}+frac{2k^3}{3}-frac{k}{105}
        =frac{15k^7+21k^5+70k^3-k}{3cdot 5cdot 7}
        $$

        To prove this is an integer we need that:
        $$15k^7+21k^5+70k^3-kequiv 0 pmod{3cdot 5cdot 7}$$
        According to the Chinese Remainder Theorem, this is the case iff
        $$begin{cases}15k^7+21k^5+70k^3-kequiv 0 pmod{3} \
        15k^7+21k^5+70k^3-kequiv 0 pmod{5}\
        15k^7+21k^5+70k^3-kequiv 0 pmod{7}end{cases} iff
        begin{cases}k^3-kequiv 0 pmod{3} \
        k^5-kequiv 0 pmod{5}\
        k^7-kequiv 0 pmod{7}end{cases}$$

        Fermat's Little Theorem says that $k^pequiv k pmod{p}$ for any prime $p$ and integer $k$.



        Therefore the original expression is an integer.







        share|cite|improve this answer














        share|cite|improve this answer



        share|cite|improve this answer








        edited Jan 16 at 18:42

























        answered Jan 16 at 17:11









        Klaas van AarsenKlaas van Aarsen

        4,3421822




        4,3421822








        • 1




          $begingroup$
          Nice. You are a fast writer!
          $endgroup$
          – mathreadler
          Jan 16 at 17:12






        • 1




          $begingroup$
          Excellent answer!
          $endgroup$
          – manooooh
          Jan 16 at 17:15










        • $begingroup$
          Fermat's Little Theorem says that if $p$ is prime and $k$ is an integer, then we have $k^p=k$ (mod $p$). How does it follow from $k^p=k$ (mod $p$) that $k$ is an integer?
          $endgroup$
          – EuxhenH
          Jan 16 at 17:19










        • $begingroup$
          It doesn't @EuxhenH. I just stated the result first and the conditions afterwards. It's exactly how you say it is.
          $endgroup$
          – Klaas van Aarsen
          Jan 16 at 17:22










        • $begingroup$
          @EuxhenH an integer raised to an integer is always an integer because integers are closed under multiplication and addition.
          $endgroup$
          – mathreadler
          Jan 16 at 17:22
















        • 1




          $begingroup$
          Nice. You are a fast writer!
          $endgroup$
          – mathreadler
          Jan 16 at 17:12






        • 1




          $begingroup$
          Excellent answer!
          $endgroup$
          – manooooh
          Jan 16 at 17:15










        • $begingroup$
          Fermat's Little Theorem says that if $p$ is prime and $k$ is an integer, then we have $k^p=k$ (mod $p$). How does it follow from $k^p=k$ (mod $p$) that $k$ is an integer?
          $endgroup$
          – EuxhenH
          Jan 16 at 17:19










        • $begingroup$
          It doesn't @EuxhenH. I just stated the result first and the conditions afterwards. It's exactly how you say it is.
          $endgroup$
          – Klaas van Aarsen
          Jan 16 at 17:22










        • $begingroup$
          @EuxhenH an integer raised to an integer is always an integer because integers are closed under multiplication and addition.
          $endgroup$
          – mathreadler
          Jan 16 at 17:22










        1




        1




        $begingroup$
        Nice. You are a fast writer!
        $endgroup$
        – mathreadler
        Jan 16 at 17:12




        $begingroup$
        Nice. You are a fast writer!
        $endgroup$
        – mathreadler
        Jan 16 at 17:12




        1




        1




        $begingroup$
        Excellent answer!
        $endgroup$
        – manooooh
        Jan 16 at 17:15




        $begingroup$
        Excellent answer!
        $endgroup$
        – manooooh
        Jan 16 at 17:15












        $begingroup$
        Fermat's Little Theorem says that if $p$ is prime and $k$ is an integer, then we have $k^p=k$ (mod $p$). How does it follow from $k^p=k$ (mod $p$) that $k$ is an integer?
        $endgroup$
        – EuxhenH
        Jan 16 at 17:19




        $begingroup$
        Fermat's Little Theorem says that if $p$ is prime and $k$ is an integer, then we have $k^p=k$ (mod $p$). How does it follow from $k^p=k$ (mod $p$) that $k$ is an integer?
        $endgroup$
        – EuxhenH
        Jan 16 at 17:19












        $begingroup$
        It doesn't @EuxhenH. I just stated the result first and the conditions afterwards. It's exactly how you say it is.
        $endgroup$
        – Klaas van Aarsen
        Jan 16 at 17:22




        $begingroup$
        It doesn't @EuxhenH. I just stated the result first and the conditions afterwards. It's exactly how you say it is.
        $endgroup$
        – Klaas van Aarsen
        Jan 16 at 17:22












        $begingroup$
        @EuxhenH an integer raised to an integer is always an integer because integers are closed under multiplication and addition.
        $endgroup$
        – mathreadler
        Jan 16 at 17:22






        $begingroup$
        @EuxhenH an integer raised to an integer is always an integer because integers are closed under multiplication and addition.
        $endgroup$
        – mathreadler
        Jan 16 at 17:22













        2












        $begingroup$

        Call the expression $f(k)$. As it's a degree $7$ polynomial, it obeys the recurrence
        $$sum_{j=0}^8(-1)^jbinom8jf(k-j)=0.$$
        Thus
        $$f(k)=8f(k-1)-28f(k-2)+56f(k-3)-70f(k-4)+56f(k-5)-28f(k-6)+8f(k-7)-f(k-8)$$
        so that if $f$ takes eight consecutive integer values, by induction, all subsequent
        values are integers too.






        share|cite|improve this answer









        $endgroup$













        • $begingroup$
          Very interesting. What causes the recurrence equation?
          $endgroup$
          – mathreadler
          Jan 16 at 17:20










        • $begingroup$
          @mathreadler iterated differences.
          $endgroup$
          – Lord Shark the Unknown
          Jan 16 at 17:21










        • $begingroup$
          Does it work only with 7 degree ? Please give a reference where i can find this relations !
          $endgroup$
          – M Desmond
          Jan 16 at 17:31








        • 1




          $begingroup$
          @MDesmond $Delta f(k) := f(k+1)-f(k), $ reduces the degree of the polynomial $,f(k),$ since leading terms cancel. So if $f$ has degree $n$ then $,Delta^{n+1} f(k) = 0,$ yields a recurrence for $f(k).,$ Above is the binomial expansion using $,Delta = S-1,$ where $,S,f(k) = f(k+1),$ is the shift operator. See any textbook that treats recurrences or finite differences.
          $endgroup$
          – Bill Dubuque
          Jan 16 at 20:01
















        2












        $begingroup$

        Call the expression $f(k)$. As it's a degree $7$ polynomial, it obeys the recurrence
        $$sum_{j=0}^8(-1)^jbinom8jf(k-j)=0.$$
        Thus
        $$f(k)=8f(k-1)-28f(k-2)+56f(k-3)-70f(k-4)+56f(k-5)-28f(k-6)+8f(k-7)-f(k-8)$$
        so that if $f$ takes eight consecutive integer values, by induction, all subsequent
        values are integers too.






        share|cite|improve this answer









        $endgroup$













        • $begingroup$
          Very interesting. What causes the recurrence equation?
          $endgroup$
          – mathreadler
          Jan 16 at 17:20










        • $begingroup$
          @mathreadler iterated differences.
          $endgroup$
          – Lord Shark the Unknown
          Jan 16 at 17:21










        • $begingroup$
          Does it work only with 7 degree ? Please give a reference where i can find this relations !
          $endgroup$
          – M Desmond
          Jan 16 at 17:31








        • 1




          $begingroup$
          @MDesmond $Delta f(k) := f(k+1)-f(k), $ reduces the degree of the polynomial $,f(k),$ since leading terms cancel. So if $f$ has degree $n$ then $,Delta^{n+1} f(k) = 0,$ yields a recurrence for $f(k).,$ Above is the binomial expansion using $,Delta = S-1,$ where $,S,f(k) = f(k+1),$ is the shift operator. See any textbook that treats recurrences or finite differences.
          $endgroup$
          – Bill Dubuque
          Jan 16 at 20:01














        2












        2








        2





        $begingroup$

        Call the expression $f(k)$. As it's a degree $7$ polynomial, it obeys the recurrence
        $$sum_{j=0}^8(-1)^jbinom8jf(k-j)=0.$$
        Thus
        $$f(k)=8f(k-1)-28f(k-2)+56f(k-3)-70f(k-4)+56f(k-5)-28f(k-6)+8f(k-7)-f(k-8)$$
        so that if $f$ takes eight consecutive integer values, by induction, all subsequent
        values are integers too.






        share|cite|improve this answer









        $endgroup$



        Call the expression $f(k)$. As it's a degree $7$ polynomial, it obeys the recurrence
        $$sum_{j=0}^8(-1)^jbinom8jf(k-j)=0.$$
        Thus
        $$f(k)=8f(k-1)-28f(k-2)+56f(k-3)-70f(k-4)+56f(k-5)-28f(k-6)+8f(k-7)-f(k-8)$$
        so that if $f$ takes eight consecutive integer values, by induction, all subsequent
        values are integers too.







        share|cite|improve this answer












        share|cite|improve this answer



        share|cite|improve this answer










        answered Jan 16 at 17:14









        Lord Shark the UnknownLord Shark the Unknown

        108k1162135




        108k1162135












        • $begingroup$
          Very interesting. What causes the recurrence equation?
          $endgroup$
          – mathreadler
          Jan 16 at 17:20










        • $begingroup$
          @mathreadler iterated differences.
          $endgroup$
          – Lord Shark the Unknown
          Jan 16 at 17:21










        • $begingroup$
          Does it work only with 7 degree ? Please give a reference where i can find this relations !
          $endgroup$
          – M Desmond
          Jan 16 at 17:31








        • 1




          $begingroup$
          @MDesmond $Delta f(k) := f(k+1)-f(k), $ reduces the degree of the polynomial $,f(k),$ since leading terms cancel. So if $f$ has degree $n$ then $,Delta^{n+1} f(k) = 0,$ yields a recurrence for $f(k).,$ Above is the binomial expansion using $,Delta = S-1,$ where $,S,f(k) = f(k+1),$ is the shift operator. See any textbook that treats recurrences or finite differences.
          $endgroup$
          – Bill Dubuque
          Jan 16 at 20:01


















        • $begingroup$
          Very interesting. What causes the recurrence equation?
          $endgroup$
          – mathreadler
          Jan 16 at 17:20










        • $begingroup$
          @mathreadler iterated differences.
          $endgroup$
          – Lord Shark the Unknown
          Jan 16 at 17:21










        • $begingroup$
          Does it work only with 7 degree ? Please give a reference where i can find this relations !
          $endgroup$
          – M Desmond
          Jan 16 at 17:31








        • 1




          $begingroup$
          @MDesmond $Delta f(k) := f(k+1)-f(k), $ reduces the degree of the polynomial $,f(k),$ since leading terms cancel. So if $f$ has degree $n$ then $,Delta^{n+1} f(k) = 0,$ yields a recurrence for $f(k).,$ Above is the binomial expansion using $,Delta = S-1,$ where $,S,f(k) = f(k+1),$ is the shift operator. See any textbook that treats recurrences or finite differences.
          $endgroup$
          – Bill Dubuque
          Jan 16 at 20:01
















        $begingroup$
        Very interesting. What causes the recurrence equation?
        $endgroup$
        – mathreadler
        Jan 16 at 17:20




        $begingroup$
        Very interesting. What causes the recurrence equation?
        $endgroup$
        – mathreadler
        Jan 16 at 17:20












        $begingroup$
        @mathreadler iterated differences.
        $endgroup$
        – Lord Shark the Unknown
        Jan 16 at 17:21




        $begingroup$
        @mathreadler iterated differences.
        $endgroup$
        – Lord Shark the Unknown
        Jan 16 at 17:21












        $begingroup$
        Does it work only with 7 degree ? Please give a reference where i can find this relations !
        $endgroup$
        – M Desmond
        Jan 16 at 17:31






        $begingroup$
        Does it work only with 7 degree ? Please give a reference where i can find this relations !
        $endgroup$
        – M Desmond
        Jan 16 at 17:31






        1




        1




        $begingroup$
        @MDesmond $Delta f(k) := f(k+1)-f(k), $ reduces the degree of the polynomial $,f(k),$ since leading terms cancel. So if $f$ has degree $n$ then $,Delta^{n+1} f(k) = 0,$ yields a recurrence for $f(k).,$ Above is the binomial expansion using $,Delta = S-1,$ where $,S,f(k) = f(k+1),$ is the shift operator. See any textbook that treats recurrences or finite differences.
        $endgroup$
        – Bill Dubuque
        Jan 16 at 20:01




        $begingroup$
        @MDesmond $Delta f(k) := f(k+1)-f(k), $ reduces the degree of the polynomial $,f(k),$ since leading terms cancel. So if $f$ has degree $n$ then $,Delta^{n+1} f(k) = 0,$ yields a recurrence for $f(k).,$ Above is the binomial expansion using $,Delta = S-1,$ where $,S,f(k) = f(k+1),$ is the shift operator. See any textbook that treats recurrences or finite differences.
        $endgroup$
        – Bill Dubuque
        Jan 16 at 20:01











        2












        $begingroup$

        hint...if you only want to use induction, let $$f(k)=15k^7+21k^5+70k^3-k$$
        and consider $$f(k+1)-f(k)=$$



        For the induction step you have to show this is divisible by $105$



        So, for example, $$(k+1)^7-k^7=7N+1$$ where $N$ is an integer, etc...



        Can you finish?






        share|cite|improve this answer









        $endgroup$


















          2












          $begingroup$

          hint...if you only want to use induction, let $$f(k)=15k^7+21k^5+70k^3-k$$
          and consider $$f(k+1)-f(k)=$$



          For the induction step you have to show this is divisible by $105$



          So, for example, $$(k+1)^7-k^7=7N+1$$ where $N$ is an integer, etc...



          Can you finish?






          share|cite|improve this answer









          $endgroup$
















            2












            2








            2





            $begingroup$

            hint...if you only want to use induction, let $$f(k)=15k^7+21k^5+70k^3-k$$
            and consider $$f(k+1)-f(k)=$$



            For the induction step you have to show this is divisible by $105$



            So, for example, $$(k+1)^7-k^7=7N+1$$ where $N$ is an integer, etc...



            Can you finish?






            share|cite|improve this answer









            $endgroup$



            hint...if you only want to use induction, let $$f(k)=15k^7+21k^5+70k^3-k$$
            and consider $$f(k+1)-f(k)=$$



            For the induction step you have to show this is divisible by $105$



            So, for example, $$(k+1)^7-k^7=7N+1$$ where $N$ is an integer, etc...



            Can you finish?







            share|cite|improve this answer












            share|cite|improve this answer



            share|cite|improve this answer










            answered Jan 16 at 17:20









            David QuinnDavid Quinn

            24.1k21141




            24.1k21141























                2












                $begingroup$

                You can use the binomial transform to prove that



                $$frac{k^7}{7}+frac{k^5}{5}+frac{2k^3}{3}-frac{k}{105}
                \={kchoose1}+28{kchoose2}+292{kchoose3}+1248{kchoose4}+2424{kchoose5}+2160{kchoose6}+720{kchoose7}$$






                share|cite|improve this answer









                $endgroup$


















                  2












                  $begingroup$

                  You can use the binomial transform to prove that



                  $$frac{k^7}{7}+frac{k^5}{5}+frac{2k^3}{3}-frac{k}{105}
                  \={kchoose1}+28{kchoose2}+292{kchoose3}+1248{kchoose4}+2424{kchoose5}+2160{kchoose6}+720{kchoose7}$$






                  share|cite|improve this answer









                  $endgroup$
















                    2












                    2








                    2





                    $begingroup$

                    You can use the binomial transform to prove that



                    $$frac{k^7}{7}+frac{k^5}{5}+frac{2k^3}{3}-frac{k}{105}
                    \={kchoose1}+28{kchoose2}+292{kchoose3}+1248{kchoose4}+2424{kchoose5}+2160{kchoose6}+720{kchoose7}$$






                    share|cite|improve this answer









                    $endgroup$



                    You can use the binomial transform to prove that



                    $$frac{k^7}{7}+frac{k^5}{5}+frac{2k^3}{3}-frac{k}{105}
                    \={kchoose1}+28{kchoose2}+292{kchoose3}+1248{kchoose4}+2424{kchoose5}+2160{kchoose6}+720{kchoose7}$$







                    share|cite|improve this answer












                    share|cite|improve this answer



                    share|cite|improve this answer










                    answered Jan 16 at 17:26









                    Jean-Claude ArbautJean-Claude Arbaut

                    14.9k63464




                    14.9k63464























                        0












                        $begingroup$

                        Base case for $k=1$: $$frac{1^7}{7}+frac{1^5}{5}+frac{2*1^3}{3}-frac{1}{105}=1$$



                        Now, assume for some k that $frac{k^7}{7}+frac{k^5}{5}+frac{2k^3}{3}-frac{k}{105}$ is indeed in integer.



                        Then $$frac{(k+1)^7}{7}+frac{(k+1)^5}{5}+frac{2(k+1)^3}{3}-frac{k+1}{105}=\frac{sum_{i=0}^7binom{7}{i}k^i}{7}+frac{sum_{i=0}^5binom{5}{i}k^i}{5}+2frac{sum_{i=0}^3binom{3}{i}k^i}{3}-frac{k+1}{105}=$$



                        Extracting the highest indexed term from each sum (and the $-frac{k}{105}$ at the end): $$frac{k^7}{7}+frac{k^5}{5}+frac{2k^3}{3}-frac{k}{105}+frac{sum_{i=0}^6binom{7}{i}k^i}{7}+frac{sum_{i=0}^4binom{5}{i}k^i}{5}+2frac{sum_{i=0}^2binom{3}{i}k^i}{3}-frac{1}{105}$$



                        By the induction hypothesis, the sum of the first four terms is an integer so, if we can show the rest of the above sum is an integer, we will be done. Use the fact that, for any prime, $p$, $p|binom{p}{k}$ where $1leq kleq p-1$. This is because $$binom{p}{k}=frac{p(p-1)...(p-k+1)}{k(k-1)...1}$$



                        $p$ divides the numerator but not the denominator (as $1leq kleq p-1$) so $p|binom{p}{k}$



                        So each term in the remaining sum with index $i$ $geq1$ and $leq p-1$ ($p$ being the respective prime in each sum) is divisible by the corresponding $p$ in the denominator and produces an integer. The only non-integer terms left will be the ones at $i=0$, i.e. $$frac{binom{7}{0}k^0}{7}+frac{binom{5}{0}k^0}{5}+frac{2binom{3}{0}k^0}{3}-frac{1}{105}=frac{1}{7}+frac{1}{5}+frac{2}{3}-frac{1}{105}=1$$



                        So $frac{(k+1)^7}{7}+frac{(k+1)^5}{5}+frac{2(k+1)^3}{3}-frac{k+1}{105}$ is a sum of integers making it an integer.






                        share|cite|improve this answer









                        $endgroup$


















                          0












                          $begingroup$

                          Base case for $k=1$: $$frac{1^7}{7}+frac{1^5}{5}+frac{2*1^3}{3}-frac{1}{105}=1$$



                          Now, assume for some k that $frac{k^7}{7}+frac{k^5}{5}+frac{2k^3}{3}-frac{k}{105}$ is indeed in integer.



                          Then $$frac{(k+1)^7}{7}+frac{(k+1)^5}{5}+frac{2(k+1)^3}{3}-frac{k+1}{105}=\frac{sum_{i=0}^7binom{7}{i}k^i}{7}+frac{sum_{i=0}^5binom{5}{i}k^i}{5}+2frac{sum_{i=0}^3binom{3}{i}k^i}{3}-frac{k+1}{105}=$$



                          Extracting the highest indexed term from each sum (and the $-frac{k}{105}$ at the end): $$frac{k^7}{7}+frac{k^5}{5}+frac{2k^3}{3}-frac{k}{105}+frac{sum_{i=0}^6binom{7}{i}k^i}{7}+frac{sum_{i=0}^4binom{5}{i}k^i}{5}+2frac{sum_{i=0}^2binom{3}{i}k^i}{3}-frac{1}{105}$$



                          By the induction hypothesis, the sum of the first four terms is an integer so, if we can show the rest of the above sum is an integer, we will be done. Use the fact that, for any prime, $p$, $p|binom{p}{k}$ where $1leq kleq p-1$. This is because $$binom{p}{k}=frac{p(p-1)...(p-k+1)}{k(k-1)...1}$$



                          $p$ divides the numerator but not the denominator (as $1leq kleq p-1$) so $p|binom{p}{k}$



                          So each term in the remaining sum with index $i$ $geq1$ and $leq p-1$ ($p$ being the respective prime in each sum) is divisible by the corresponding $p$ in the denominator and produces an integer. The only non-integer terms left will be the ones at $i=0$, i.e. $$frac{binom{7}{0}k^0}{7}+frac{binom{5}{0}k^0}{5}+frac{2binom{3}{0}k^0}{3}-frac{1}{105}=frac{1}{7}+frac{1}{5}+frac{2}{3}-frac{1}{105}=1$$



                          So $frac{(k+1)^7}{7}+frac{(k+1)^5}{5}+frac{2(k+1)^3}{3}-frac{k+1}{105}$ is a sum of integers making it an integer.






                          share|cite|improve this answer









                          $endgroup$
















                            0












                            0








                            0





                            $begingroup$

                            Base case for $k=1$: $$frac{1^7}{7}+frac{1^5}{5}+frac{2*1^3}{3}-frac{1}{105}=1$$



                            Now, assume for some k that $frac{k^7}{7}+frac{k^5}{5}+frac{2k^3}{3}-frac{k}{105}$ is indeed in integer.



                            Then $$frac{(k+1)^7}{7}+frac{(k+1)^5}{5}+frac{2(k+1)^3}{3}-frac{k+1}{105}=\frac{sum_{i=0}^7binom{7}{i}k^i}{7}+frac{sum_{i=0}^5binom{5}{i}k^i}{5}+2frac{sum_{i=0}^3binom{3}{i}k^i}{3}-frac{k+1}{105}=$$



                            Extracting the highest indexed term from each sum (and the $-frac{k}{105}$ at the end): $$frac{k^7}{7}+frac{k^5}{5}+frac{2k^3}{3}-frac{k}{105}+frac{sum_{i=0}^6binom{7}{i}k^i}{7}+frac{sum_{i=0}^4binom{5}{i}k^i}{5}+2frac{sum_{i=0}^2binom{3}{i}k^i}{3}-frac{1}{105}$$



                            By the induction hypothesis, the sum of the first four terms is an integer so, if we can show the rest of the above sum is an integer, we will be done. Use the fact that, for any prime, $p$, $p|binom{p}{k}$ where $1leq kleq p-1$. This is because $$binom{p}{k}=frac{p(p-1)...(p-k+1)}{k(k-1)...1}$$



                            $p$ divides the numerator but not the denominator (as $1leq kleq p-1$) so $p|binom{p}{k}$



                            So each term in the remaining sum with index $i$ $geq1$ and $leq p-1$ ($p$ being the respective prime in each sum) is divisible by the corresponding $p$ in the denominator and produces an integer. The only non-integer terms left will be the ones at $i=0$, i.e. $$frac{binom{7}{0}k^0}{7}+frac{binom{5}{0}k^0}{5}+frac{2binom{3}{0}k^0}{3}-frac{1}{105}=frac{1}{7}+frac{1}{5}+frac{2}{3}-frac{1}{105}=1$$



                            So $frac{(k+1)^7}{7}+frac{(k+1)^5}{5}+frac{2(k+1)^3}{3}-frac{k+1}{105}$ is a sum of integers making it an integer.






                            share|cite|improve this answer









                            $endgroup$



                            Base case for $k=1$: $$frac{1^7}{7}+frac{1^5}{5}+frac{2*1^3}{3}-frac{1}{105}=1$$



                            Now, assume for some k that $frac{k^7}{7}+frac{k^5}{5}+frac{2k^3}{3}-frac{k}{105}$ is indeed in integer.



                            Then $$frac{(k+1)^7}{7}+frac{(k+1)^5}{5}+frac{2(k+1)^3}{3}-frac{k+1}{105}=\frac{sum_{i=0}^7binom{7}{i}k^i}{7}+frac{sum_{i=0}^5binom{5}{i}k^i}{5}+2frac{sum_{i=0}^3binom{3}{i}k^i}{3}-frac{k+1}{105}=$$



                            Extracting the highest indexed term from each sum (and the $-frac{k}{105}$ at the end): $$frac{k^7}{7}+frac{k^5}{5}+frac{2k^3}{3}-frac{k}{105}+frac{sum_{i=0}^6binom{7}{i}k^i}{7}+frac{sum_{i=0}^4binom{5}{i}k^i}{5}+2frac{sum_{i=0}^2binom{3}{i}k^i}{3}-frac{1}{105}$$



                            By the induction hypothesis, the sum of the first four terms is an integer so, if we can show the rest of the above sum is an integer, we will be done. Use the fact that, for any prime, $p$, $p|binom{p}{k}$ where $1leq kleq p-1$. This is because $$binom{p}{k}=frac{p(p-1)...(p-k+1)}{k(k-1)...1}$$



                            $p$ divides the numerator but not the denominator (as $1leq kleq p-1$) so $p|binom{p}{k}$



                            So each term in the remaining sum with index $i$ $geq1$ and $leq p-1$ ($p$ being the respective prime in each sum) is divisible by the corresponding $p$ in the denominator and produces an integer. The only non-integer terms left will be the ones at $i=0$, i.e. $$frac{binom{7}{0}k^0}{7}+frac{binom{5}{0}k^0}{5}+frac{2binom{3}{0}k^0}{3}-frac{1}{105}=frac{1}{7}+frac{1}{5}+frac{2}{3}-frac{1}{105}=1$$



                            So $frac{(k+1)^7}{7}+frac{(k+1)^5}{5}+frac{2(k+1)^3}{3}-frac{k+1}{105}$ is a sum of integers making it an integer.







                            share|cite|improve this answer












                            share|cite|improve this answer



                            share|cite|improve this answer










                            answered Jan 16 at 17:48









                            Cardioid_Ass_22Cardioid_Ass_22

                            47815




                            47815























                                0












                                $begingroup$

                                Because
                                $$frac{k^7}{7}+frac{k^5}{5}+frac{2k^3}{3}-frac{k}{105}=frac{k^7-k}{7}+frac{k^5-k}{5}+frac{2(k^3-k)}{3}+k.$$






                                share|cite|improve this answer









                                $endgroup$













                                • $begingroup$
                                  Same as my answer after dividing by $,3cdot 5cdot 7 $
                                  $endgroup$
                                  – Bill Dubuque
                                  Jan 16 at 19:18












                                • $begingroup$
                                  They are similar but my equality we can see immediately. See please better that I wrote.
                                  $endgroup$
                                  – Michael Rozenberg
                                  Jan 16 at 19:20










                                • $begingroup$
                                  In my experience most students don't "immediately" see such fraction expansions.. Rather they first put it over a common denominator, as I did. From that it is easy to read off the fraction expansion (but there is no need to rewrite the key (Fernat) divisibilities in fraction form).
                                  $endgroup$
                                  – Bill Dubuque
                                  Jan 16 at 19:28












                                • $begingroup$
                                  I think you see that $-frac{1}{105}=-frac{1}{7}-frac{1}{5}-frac{2}{3}+1$. If so, we are done! I think, it's much more better than your writing.
                                  $endgroup$
                                  – Michael Rozenberg
                                  Jan 16 at 19:31












                                • $begingroup$
                                  How do you propose that one "sees" things like that in general?
                                  $endgroup$
                                  – Bill Dubuque
                                  Jan 16 at 19:32
















                                0












                                $begingroup$

                                Because
                                $$frac{k^7}{7}+frac{k^5}{5}+frac{2k^3}{3}-frac{k}{105}=frac{k^7-k}{7}+frac{k^5-k}{5}+frac{2(k^3-k)}{3}+k.$$






                                share|cite|improve this answer









                                $endgroup$













                                • $begingroup$
                                  Same as my answer after dividing by $,3cdot 5cdot 7 $
                                  $endgroup$
                                  – Bill Dubuque
                                  Jan 16 at 19:18












                                • $begingroup$
                                  They are similar but my equality we can see immediately. See please better that I wrote.
                                  $endgroup$
                                  – Michael Rozenberg
                                  Jan 16 at 19:20










                                • $begingroup$
                                  In my experience most students don't "immediately" see such fraction expansions.. Rather they first put it over a common denominator, as I did. From that it is easy to read off the fraction expansion (but there is no need to rewrite the key (Fernat) divisibilities in fraction form).
                                  $endgroup$
                                  – Bill Dubuque
                                  Jan 16 at 19:28












                                • $begingroup$
                                  I think you see that $-frac{1}{105}=-frac{1}{7}-frac{1}{5}-frac{2}{3}+1$. If so, we are done! I think, it's much more better than your writing.
                                  $endgroup$
                                  – Michael Rozenberg
                                  Jan 16 at 19:31












                                • $begingroup$
                                  How do you propose that one "sees" things like that in general?
                                  $endgroup$
                                  – Bill Dubuque
                                  Jan 16 at 19:32














                                0












                                0








                                0





                                $begingroup$

                                Because
                                $$frac{k^7}{7}+frac{k^5}{5}+frac{2k^3}{3}-frac{k}{105}=frac{k^7-k}{7}+frac{k^5-k}{5}+frac{2(k^3-k)}{3}+k.$$






                                share|cite|improve this answer









                                $endgroup$



                                Because
                                $$frac{k^7}{7}+frac{k^5}{5}+frac{2k^3}{3}-frac{k}{105}=frac{k^7-k}{7}+frac{k^5-k}{5}+frac{2(k^3-k)}{3}+k.$$







                                share|cite|improve this answer












                                share|cite|improve this answer



                                share|cite|improve this answer










                                answered Jan 16 at 19:13









                                Michael RozenbergMichael Rozenberg

                                110k1896201




                                110k1896201












                                • $begingroup$
                                  Same as my answer after dividing by $,3cdot 5cdot 7 $
                                  $endgroup$
                                  – Bill Dubuque
                                  Jan 16 at 19:18












                                • $begingroup$
                                  They are similar but my equality we can see immediately. See please better that I wrote.
                                  $endgroup$
                                  – Michael Rozenberg
                                  Jan 16 at 19:20










                                • $begingroup$
                                  In my experience most students don't "immediately" see such fraction expansions.. Rather they first put it over a common denominator, as I did. From that it is easy to read off the fraction expansion (but there is no need to rewrite the key (Fernat) divisibilities in fraction form).
                                  $endgroup$
                                  – Bill Dubuque
                                  Jan 16 at 19:28












                                • $begingroup$
                                  I think you see that $-frac{1}{105}=-frac{1}{7}-frac{1}{5}-frac{2}{3}+1$. If so, we are done! I think, it's much more better than your writing.
                                  $endgroup$
                                  – Michael Rozenberg
                                  Jan 16 at 19:31












                                • $begingroup$
                                  How do you propose that one "sees" things like that in general?
                                  $endgroup$
                                  – Bill Dubuque
                                  Jan 16 at 19:32


















                                • $begingroup$
                                  Same as my answer after dividing by $,3cdot 5cdot 7 $
                                  $endgroup$
                                  – Bill Dubuque
                                  Jan 16 at 19:18












                                • $begingroup$
                                  They are similar but my equality we can see immediately. See please better that I wrote.
                                  $endgroup$
                                  – Michael Rozenberg
                                  Jan 16 at 19:20










                                • $begingroup$
                                  In my experience most students don't "immediately" see such fraction expansions.. Rather they first put it over a common denominator, as I did. From that it is easy to read off the fraction expansion (but there is no need to rewrite the key (Fernat) divisibilities in fraction form).
                                  $endgroup$
                                  – Bill Dubuque
                                  Jan 16 at 19:28












                                • $begingroup$
                                  I think you see that $-frac{1}{105}=-frac{1}{7}-frac{1}{5}-frac{2}{3}+1$. If so, we are done! I think, it's much more better than your writing.
                                  $endgroup$
                                  – Michael Rozenberg
                                  Jan 16 at 19:31












                                • $begingroup$
                                  How do you propose that one "sees" things like that in general?
                                  $endgroup$
                                  – Bill Dubuque
                                  Jan 16 at 19:32
















                                $begingroup$
                                Same as my answer after dividing by $,3cdot 5cdot 7 $
                                $endgroup$
                                – Bill Dubuque
                                Jan 16 at 19:18






                                $begingroup$
                                Same as my answer after dividing by $,3cdot 5cdot 7 $
                                $endgroup$
                                – Bill Dubuque
                                Jan 16 at 19:18














                                $begingroup$
                                They are similar but my equality we can see immediately. See please better that I wrote.
                                $endgroup$
                                – Michael Rozenberg
                                Jan 16 at 19:20




                                $begingroup$
                                They are similar but my equality we can see immediately. See please better that I wrote.
                                $endgroup$
                                – Michael Rozenberg
                                Jan 16 at 19:20












                                $begingroup$
                                In my experience most students don't "immediately" see such fraction expansions.. Rather they first put it over a common denominator, as I did. From that it is easy to read off the fraction expansion (but there is no need to rewrite the key (Fernat) divisibilities in fraction form).
                                $endgroup$
                                – Bill Dubuque
                                Jan 16 at 19:28






                                $begingroup$
                                In my experience most students don't "immediately" see such fraction expansions.. Rather they first put it over a common denominator, as I did. From that it is easy to read off the fraction expansion (but there is no need to rewrite the key (Fernat) divisibilities in fraction form).
                                $endgroup$
                                – Bill Dubuque
                                Jan 16 at 19:28














                                $begingroup$
                                I think you see that $-frac{1}{105}=-frac{1}{7}-frac{1}{5}-frac{2}{3}+1$. If so, we are done! I think, it's much more better than your writing.
                                $endgroup$
                                – Michael Rozenberg
                                Jan 16 at 19:31






                                $begingroup$
                                I think you see that $-frac{1}{105}=-frac{1}{7}-frac{1}{5}-frac{2}{3}+1$. If so, we are done! I think, it's much more better than your writing.
                                $endgroup$
                                – Michael Rozenberg
                                Jan 16 at 19:31














                                $begingroup$
                                How do you propose that one "sees" things like that in general?
                                $endgroup$
                                – Bill Dubuque
                                Jan 16 at 19:32




                                $begingroup$
                                How do you propose that one "sees" things like that in general?
                                $endgroup$
                                – Bill Dubuque
                                Jan 16 at 19:32











                                0












                                $begingroup$

                                Hint $ $ Note that $ 3!cdot!5!cdot!7mid overbrace{3!cdot! 5, (color{#c00}{k^7!-!k})+ 3!cdot! 7, (color{#c00}{k^5!-!k})- 5!cdot! 7 (color{#c00}{k^3!-!k})+ 3!cdot! 5cdot! 7, k^3}^{Large{rm sum = this/(3cdot 5cdot 7)}}, $ by $,rmoverbrace{little color{#c00}{Fermat}}^{Large p mid color{#c00}{k^p-k}}$



                                Remark $ $ More generally this shows that if $,p,q,r,$ are primes and $,a,b,c,k,$ are integers



                                $$quad pqr,mid, aqr,(k^p!-!k)+bpr,(k^q!-!k)+cpq,(k^r!-!k)$$






                                share|cite|improve this answer











                                $endgroup$













                                • $begingroup$
                                  Note that we don't actually have to recognize the above form to prove it is $equiv 0pmod{p},$ for $,p = 3,5,7,,$ since simply computing it $!bmod p,$ using $,k^pequiv k,$ easily proves it is $equiv 0, $ But I showed the form of the expression to reveal how it was constructed.
                                  $endgroup$
                                  – Bill Dubuque
                                  Jan 16 at 19:35
















                                0












                                $begingroup$

                                Hint $ $ Note that $ 3!cdot!5!cdot!7mid overbrace{3!cdot! 5, (color{#c00}{k^7!-!k})+ 3!cdot! 7, (color{#c00}{k^5!-!k})- 5!cdot! 7 (color{#c00}{k^3!-!k})+ 3!cdot! 5cdot! 7, k^3}^{Large{rm sum = this/(3cdot 5cdot 7)}}, $ by $,rmoverbrace{little color{#c00}{Fermat}}^{Large p mid color{#c00}{k^p-k}}$



                                Remark $ $ More generally this shows that if $,p,q,r,$ are primes and $,a,b,c,k,$ are integers



                                $$quad pqr,mid, aqr,(k^p!-!k)+bpr,(k^q!-!k)+cpq,(k^r!-!k)$$






                                share|cite|improve this answer











                                $endgroup$













                                • $begingroup$
                                  Note that we don't actually have to recognize the above form to prove it is $equiv 0pmod{p},$ for $,p = 3,5,7,,$ since simply computing it $!bmod p,$ using $,k^pequiv k,$ easily proves it is $equiv 0, $ But I showed the form of the expression to reveal how it was constructed.
                                  $endgroup$
                                  – Bill Dubuque
                                  Jan 16 at 19:35














                                0












                                0








                                0





                                $begingroup$

                                Hint $ $ Note that $ 3!cdot!5!cdot!7mid overbrace{3!cdot! 5, (color{#c00}{k^7!-!k})+ 3!cdot! 7, (color{#c00}{k^5!-!k})- 5!cdot! 7 (color{#c00}{k^3!-!k})+ 3!cdot! 5cdot! 7, k^3}^{Large{rm sum = this/(3cdot 5cdot 7)}}, $ by $,rmoverbrace{little color{#c00}{Fermat}}^{Large p mid color{#c00}{k^p-k}}$



                                Remark $ $ More generally this shows that if $,p,q,r,$ are primes and $,a,b,c,k,$ are integers



                                $$quad pqr,mid, aqr,(k^p!-!k)+bpr,(k^q!-!k)+cpq,(k^r!-!k)$$






                                share|cite|improve this answer











                                $endgroup$



                                Hint $ $ Note that $ 3!cdot!5!cdot!7mid overbrace{3!cdot! 5, (color{#c00}{k^7!-!k})+ 3!cdot! 7, (color{#c00}{k^5!-!k})- 5!cdot! 7 (color{#c00}{k^3!-!k})+ 3!cdot! 5cdot! 7, k^3}^{Large{rm sum = this/(3cdot 5cdot 7)}}, $ by $,rmoverbrace{little color{#c00}{Fermat}}^{Large p mid color{#c00}{k^p-k}}$



                                Remark $ $ More generally this shows that if $,p,q,r,$ are primes and $,a,b,c,k,$ are integers



                                $$quad pqr,mid, aqr,(k^p!-!k)+bpr,(k^q!-!k)+cpq,(k^r!-!k)$$







                                share|cite|improve this answer














                                share|cite|improve this answer



                                share|cite|improve this answer








                                edited Jan 16 at 20:26

























                                answered Jan 16 at 18:05









                                Bill DubuqueBill Dubuque

                                213k29196654




                                213k29196654












                                • $begingroup$
                                  Note that we don't actually have to recognize the above form to prove it is $equiv 0pmod{p},$ for $,p = 3,5,7,,$ since simply computing it $!bmod p,$ using $,k^pequiv k,$ easily proves it is $equiv 0, $ But I showed the form of the expression to reveal how it was constructed.
                                  $endgroup$
                                  – Bill Dubuque
                                  Jan 16 at 19:35


















                                • $begingroup$
                                  Note that we don't actually have to recognize the above form to prove it is $equiv 0pmod{p},$ for $,p = 3,5,7,,$ since simply computing it $!bmod p,$ using $,k^pequiv k,$ easily proves it is $equiv 0, $ But I showed the form of the expression to reveal how it was constructed.
                                  $endgroup$
                                  – Bill Dubuque
                                  Jan 16 at 19:35
















                                $begingroup$
                                Note that we don't actually have to recognize the above form to prove it is $equiv 0pmod{p},$ for $,p = 3,5,7,,$ since simply computing it $!bmod p,$ using $,k^pequiv k,$ easily proves it is $equiv 0, $ But I showed the form of the expression to reveal how it was constructed.
                                $endgroup$
                                – Bill Dubuque
                                Jan 16 at 19:35




                                $begingroup$
                                Note that we don't actually have to recognize the above form to prove it is $equiv 0pmod{p},$ for $,p = 3,5,7,,$ since simply computing it $!bmod p,$ using $,k^pequiv k,$ easily proves it is $equiv 0, $ But I showed the form of the expression to reveal how it was constructed.
                                $endgroup$
                                – Bill Dubuque
                                Jan 16 at 19:35



                                Popular posts from this blog

                                Human spaceflight

                                Can not write log (Is /dev/pts mounted?) - openpty in Ubuntu-on-Windows?

                                File:DeusFollowingSea.jpg